Composition of a Continuous and Measurable Function

Click For Summary
The discussion centers on whether the composition of a measurable function f and a continuous function g, denoted as f ∘ g, is necessarily measurable. It is established that while f is measurable, the continuity of g does not guarantee that the composition retains measurability. The argument suggests that the image of a measurable set under a continuous function may not be measurable, indicating that the statement is not universally true. A specific example is referenced where a continuous, strictly increasing function is used alongside a non-Lebesgue measurable set to demonstrate that the composition can fail to be measurable. Overall, the conclusion drawn is that the composition f ∘ g is not necessarily measurable.
Bashyboy
Messages
1,419
Reaction score
5

Homework Statement


Suppose that ##f## and ##g## are real-valued functions defined on all of ##\Bbb{R}##,##f## is measurable, and ##g## is continuous. Is the composition ##f \circ g## necessarily measurable?

Homework Equations

The Attempt at a Solution



Let ##c \in \Bbb{R}## be arbitrary. Then ##\{x \in \Bbb{R} ~|~ f(g(x)) > c \} = \{x \in g( \Bbb{R}) ~|~ f(x) > c \} \subseteq \{x \in \Bbb{R} ~|~ f(x) > c\}##, where the last set is measurable by our hypothesis. If ##g## were surjective, the first set would certainly be measurable, independently of whether ##g## is continuous, since equality would hold with the last set. This suggests that the statement isn't necessarily true. Another way would be to argue that ##g(\Bbb{R})## is a measurable, but apparently the continuous image of a measurable set isn't always measurable. So I suspect the statement isn't necessarily true, but I am having trouble finding a counterexample. I could use some help.
 
Physics news on Phys.org
Hold on! I am pretty certain ##\{x \in \Bbb{R} ~|~ f(g(x)) > c \} = \{x \in g( \Bbb{R}) ~|~ f(x) > c \}## is false. However, I still believe that the theorem isn't necessarily true.
 
I'm not really fit in the dungeon of topological, pathological examples, but I've seen an example which showed that the composition of two Lebesgue measurable functions isn't necessarily Lebesgue measurable. The author constructs a continuous, ##1:1##, strictly increasing function ##g## on ##[0,1]## and with the help of a nowhere dense perfect subset ##P##, ##f=\chi_A## where ##A=g(S)## and ##S \subseteq P## is a non Lebesgue measurable subset, ##f \circ g## does the job.
 
Question: A clock's minute hand has length 4 and its hour hand has length 3. What is the distance between the tips at the moment when it is increasing most rapidly?(Putnam Exam Question) Answer: Making assumption that both the hands moves at constant angular velocities, the answer is ## \sqrt{7} .## But don't you think this assumption is somewhat doubtful and wrong?

Similar threads

Replies
16
Views
2K
Replies
14
Views
3K
  • · Replies 1 ·
Replies
1
Views
2K
  • · Replies 1 ·
Replies
1
Views
2K
Replies
9
Views
3K
  • · Replies 5 ·
Replies
5
Views
2K
  • · Replies 7 ·
Replies
7
Views
3K
  • · Replies 1 ·
Replies
1
Views
2K
  • · Replies 5 ·
Replies
5
Views
3K
  • · Replies 11 ·
Replies
11
Views
2K